Evaluate integral using Green Theorem

Click For Summary
The discussion focuses on using Green's Theorem to evaluate an integral involving e^(y^2). Participants suggest changing the order of integration and emphasize the importance of adjusting the limits accordingly, ensuring they depend solely on y. One user confirms that after changing the integration order, their result appears correct. They verify their answer using Mathway, which supports the accuracy of their solution. The conversation highlights the significance of proper parametrization and limit adjustments in integral evaluation.
daphnelee-mh
Messages
62
Reaction score
4
Homework Statement
(question is attached below)
Relevant Equations
∮Pdx+Qdy=∬ [ (∂Q/∂x)-(∂P/∂y)]dA
1593773577298.png

I got stuck here, how to integrate e^(y^2), I searched but it's something like error function
 
Physics news on Phys.org
Try changing the order of integration.
 
  • Like
Likes daphnelee-mh
But you usually parametrize the curve along which you integrate. Edit: Change the limits of integration accordingly when you change the order of integration. These limits should be a function of y alone and not of x. Does that work?
 
Last edited:
WWGD said:
But you usually parametrize the curve along which you integrate. Edit: Change the limits of integration accordingly when you change the order of integration. These limits should be a function of y alone and not of x. Does that work?
1593817335698.png

I got this answer after changed the integrating order
 
daphnelee-mh said:
View attachment 265800
I got this answer after changed the integrating order
Looks right. Do you have a way of checking? Maybe Wolfram?
 
I checked it using Mathway just now, it's correct, thank you.
 
  • Like
Likes WWGD
Question: A clock's minute hand has length 4 and its hour hand has length 3. What is the distance between the tips at the moment when it is increasing most rapidly?(Putnam Exam Question) Answer: Making assumption that both the hands moves at constant angular velocities, the answer is ## \sqrt{7} .## But don't you think this assumption is somewhat doubtful and wrong?

Similar threads

Replies
3
Views
2K
Replies
28
Views
2K
  • · Replies 3 ·
Replies
3
Views
2K
  • · Replies 1 ·
Replies
1
Views
2K
  • · Replies 10 ·
Replies
10
Views
2K
  • · Replies 2 ·
Replies
2
Views
2K
  • · Replies 7 ·
Replies
7
Views
2K
  • · Replies 5 ·
Replies
5
Views
1K
  • · Replies 2 ·
Replies
2
Views
1K
  • · Replies 6 ·
Replies
6
Views
3K